Bất đẳng thức về trung bình cộng năm 2024

  • 1. by HEXAGON Volume 2009/....... /...... Bất đẳng thức giữa các lượng trung bình Phạm Văn Thuận Tóm tắt Trong bài viết này, chúng tôi sẽ giới thiệu về những bất đẳng thức liên hệ giữa các đại lượng trung bình cho nhiều số. Chúng tôi cũng trình bày ý nghĩa hình học của trung bình cộng, trung bình nhân, ứng dụng những bất đẳng thức vào một số bài toán thực tế. Các kỹ thuật quan trọng trong việc áp dụng bất đẳng thức giữa trung bình cộng và trung bình nhân cũng được minh họa bởi các thí dụ đa dạng. 1 Mở đầu Ngoài một số tính chất, quy tắc cơ bản trong chứng minh bất đẳng thức trên tập số thực như nhân, chia hai vế bất đẳng thức với một số, bình phương, nghịch đảo, nâng lũy thừa, lấy căn bậc n hai vế bất đẳng thức, chúng tôi lưu ý một số tính chất sau: i) Nếu a j là số lớn nhất trong các số a1 , a2 , ..., an thỏa mãn điều kiện a1 + a2 + · · · + an = k, k với k là hằng số, thì a j ≥ n . ii) Nếu hai số x, y thỏa mãn bất đẳng thức x ≥ y thì tồn tại một số t ≥ 1 nào đó thỏa mãn x = ty, hoặc x = t2 y, hoặc x = t3 y .... 2 Bất đẳng thức giữa trung bình cộng và trung bình nhân cho hai số x+ y √ Định lý 1. Với hai số thực không âm x, y, ta gọi a = 2 và g = xy lần lượt là trung bình cộng và trung bình nhân của hai số. Khi đó ta có bất đẳng thức a ≥ g. Đẳng thức xảy ra khi và chỉ khi x = y. Chứng minh. Ta có thể chứng minh bất đẳng thức trên bằng cách biến đổi đại số như sau. Ta viết bất đẳng thức về dạng x+y √ 1 √ √ − xy = ( x − y)2 ≥ 0. 2 2 Điều này hiển nhiên đúng. Phép chứng minh hoàn tất. Đẳng thức xảy ra khi và chỉ khi √ √ x = y hay x = y. Chứng minh. Bây giờ ta "diễn đạt" ý tưởng trên theo cách khác, mà từ đó ta sử dụng cho một tổng quát hoá. Thực vậy, với hai số x, y cho trước, ta luôn có thể chọn được một số nhỏ hơn (hoặc bằng) số kia. Không mất tổng quát, giả sử x ≥ y. Nên tồn tại một số thực dương t, t2 ≥ 1, sao cho x = yt2 . Thay, x = yt2 vào bất đẳng thức ta được bất đẳng thức tương đương yt2 + y ≥ yt2 y, 2 Copyright © HEXAGON
  • 2. − 1)2 ≥ 0. Điều này hiển nhiên đúng với y ≥ 0. Phép chứng minh hoàn tất. 1 Từ bất đẳng thức giữa trung bình cộng và trung bình nhân, ta có thể xây dựng được chuỗi bất đẳng thức sau cho cặp số thực không âm x, y Định lý 2. Cho hai số thực không âm x, y. Chứng minh rằng 2 √ x+y x2 + y2 min ( x, y) ≤ 1 1 ≤ xy ≤ ≤ ≤ max ( x, y). x + y 2 2 Chứng minh. Trước hết, ta chứng minh bất đẳng thức sau được gọi là bất đẳng thức giữa trung bình điều hòa và trung bình nhân. 2 √ 1 1 ≤ xy. (1) x + y 2 Lượng 1 1 được gọi là trung bình điều hòa của hai số x, y. Ta viết lại (1) dưới dạng x+y 2xy √ ≤ xy, x+y x+ y √ bất đẳng thức này tương đương với 2 ≥ xy. Đây là điều đã chứng minh. Tiếp theo, ta chứng minh bất đẳng thức giữa trung bình cộng và trung bình bậc hai sau đây. x+y x2 + y2 ≤ . 2 2 x2 + y2 Lượng 2 gọi là trung bình bậc hai của hai số x, y. Vì cả hai vế của bất đẳng thức đều dương, bình phương hai vế cho ta ( x + y) 2 x2 + y2 ≤ . 4 2 Lại áp dụng bất đẳng thức giữa trung bình cộng và trung bình nhân cho hai số x2 , y2 , ta có x2 + y2 xy ≤ 2 . Do đó ( x + y) 2 x2 + 2xy + y2 x2 + x2 + y2 + y2 x2 + y2 = ≤ = . 4 4 4 2 Phép chứng minh hoàn tất. Làm tương tự, ta thu được bất đẳng thức giữa trung bình cộng và trung bình bậc hai cho ba số không âm x, y, z. 1. Với các bài toán bất đẳng thức hai số, chẳng hạn x, y, mà vai trò giữa các biến số bình đẳng, ta luôn có thể đưa về được bất đẳng thức một biến số với phép biến đổi x/y = t2 . 2
  • 3. Giả sử x, y, z là ba số thực. Chứng minh rằng 2 2 x+y+z x2 + y2 + z2 ≤ . 3 3 Bây giờ ta sẽ mở rộng bất đẳng thức giữa trung bình cộng và trung bình nhân cho ba số. Ta phát biểu và chứng minh bất đẳng thức sau Định lý 3. Cho ba số thực không âm x, y, z. Chứng minh rằng x+y+z √ ≥ 3 xyz. (2) 3 Chứng minh. Vì vai trò các số x, y, z là bình đẳng, ta luôn có thể giả sử x = max ( x, y, z). Khi đó, hiển nhiên rằng ta có y+z x≥ . 2 Tức là, tồn tại một số dương k ≥ 1, sao cho y+z x= k3 . 2 y+ z Bằng cách thay x = 2 k3 khi đó bất đẳng thức cần chứng minh trở thành y+ z 3 2 k +y+z 3 y+z ≥ k3 yz. 3 2 √ Sử dụng bất đẳng thức giữa trung bình cộng và trung bình nhân cho hai số y, z ta có yz ≤ y+ z y+ z 2 2 , tức là yz ≤ 2 . Suy ra, theo tính chất bắc cầu ta cần chứng minh y+ z 3 2 2 k +y+z 3 y+z y+z ≥ k3 . 3 2 2 Bất đẳng thức này tương đương với y+ z 3 2 k +y+z y+z ≥ k, 3 2 2. Sử dụng liên tiếp ý tưởng này, ta có thể chứng minh bất đẳng thức giữa trung bình cộng và trung bình bậc hai tổng quát cho n số. Tức là ta có bất đẳng thức sau x1 + x2 + · · · + xn x2 + x2 + · · · + x2 1 2 n ≤ . n n Đây được coi là phép chứng minh đơn giản nhất cho bất đẳng thức này. 3
  • 4. y+z 3 (k − 3k + 2) ≥ 0. 2 Lưu ý rằng đa thức k3 − 3k + 2 có nghiệm k = 1, điều này gợi ý cho phép phân tích thành nhân tử. Ta có bất đẳng thức tương đương y+z ( k − 1) 2 ( k + 2) ≥ 0, 2 bất đẳng thức này hiển nhiên đúng với y, z ≥ 0, k ≥ 1. Phép chứng minh hoàn tất. 3 Sử dụng bất đẳng thức (2) cho bộ 1/x, 1/y, 1/z ta được bất đẳng thức giữa trung bình nhân và trung bình điều hòa cho ba số x, y, z. Thật vậy, ta có 1 1 1 x + y + z 1 1 1 3 ≥ . . , 3 x y z hay là √ 3 3 xyz ≥ 1 1 1 . x + y + z Tiếp tục sử dụng bất đẳng thức giữa trung bình cộng và trung bình nhân cho hai số ta sẽ thu được dãy bất đẳng thức cho bộ ba số x, y, z Định lý 4. Cho ba số thực không âm x, y, z. Chứng minh dãy bất đẳng thức 3 √ xy + yz + zx min ( x, y, z) ≤ 1 1 1 ≤ 3 xyz ≤ x + y + z 3 x+y+z x2 + y2 + z2 ≤ ≤ ≤ max ( x, y, z). 3 3 Một cách tự nhiên, ta nghĩ đến bất đẳng thức giữa trung bình cộng và trung bình nhân cho bốn số, năm số, và nhiều hơn nữa. Với trường hợp bốn số, ta sử dụng trực tiếp trường hợp hai số. Trường hợp năm số, ta có thể chứng minh tương tự như cách đã làm với ba số. Bài tập 1. Chứng minh rằng với mọi số thực không âm x, y thì √ 1 x+y √ 1 √ √ xy + | x − y| ≥ ≥ xy + ( x − y)2 . 2 2 4 3. Một cách chứng minh khác cho bất đẳng thức này là sử dụng hằng đẳng thức x3 + y3 + z3 1 − xyz = ( x + y + z)( x2 + y2 + z2 − xy − yz − zx). 3 3 4
  • 5. Cho hai số thực không âm x, y. Chứng minh bất đẳng thức √ x2 + y2 x+y 2−1 ≤ + | x − y|. 2 2 2 Bài tập 3. Cho hai số thực không âm x, y; gọi a và g lần lượt là trung bình cộng và trung bình nhân của hai số. Chứng minh rằng (1 + g)2 ≤ (1 + x)(1 + y) ≤ (1 + a)2 . 3 Ý nghĩa hình học của trung bình cộng, trung bình nhân Giả sử ta có một tập các hình chữ nhật có diện tích A và độ dài cạnh là x, y. Vì A = xy, nên x+ y √ √ bất đẳng thức 2 ≥ xy có ý nghĩa là hình vuông có độ dài cạnh xy phải có chu vi nhỏ nhất trong số các hình chữ nhật có cùng diện tích A. Nói cách khác, trong số tất cả các hình chữ nhật có chu vi p thì hình vuông có cạnh p/4 là hình vuông có diện tích lớn nhất. x y Hai đường tròn bán kính 2 và 2 tiếp xúc ngoài với nhau. Vẽ các bán kính của mỗi đường tròn vuông góc với tiếp tuyến chung của hai đường tròn. Khi đó, đoạn thẳng nối tâm hai đường tròn sẽ có độ dài 1 ( x + y), đoạn thẳng nối hai tiếp điểm của đường thẳng với hai 2 √ đường tròn có độ dài là xy. Tại sao? x+ y 2 xy x y √ 2 2 √ x y xy Ta có thể chứng minh điều này bằng cách sử dụng định lý Pythagore. Chú ý đẳng thức 2 2 x+y √ x−y = ( xy)2 + . 2 2 Một ý nghĩa hình học nữa liên quan đến trung bình cộng và trung bình nhân là coi 1 ( x + y) 2 √ là bán kính của một đường tròn và xy là đường cao xuất phát từ đỉnh góc vuông chắn nửa đường tròn. Một số câu hỏi tự nhiên được đặt ra là: liệu còn cách mô tả ý nghĩa hình học nào nữa về hai đại lượng trung bình trên không?; ý nghĩa hình học của trung bình điều hòa hai số là gì? 4 Ứng dụng bất đẳng thức giữa các lượng trung bình Bài toán 2. Một tấm nhôm hình vuông có cạnh bằng 50 cm. Người ta cắt ở bốn góc bốn hình vuông bằng nhau, rồi gấp tấm nhôm lại để được một cái hộp không nắp. Tính cạnh của các hình vuông bị cắt sao cho thể tích của khối hộp là lớn nhất. 5
  • 6. 50 Chứng minh. Gọi x (cm) là độ dài cạnh của hình vuông bị cắt, x phải là số dương và nhỏ hơn 25. Thể tích của khối hộp V tính theo x là V = x(50 − 2x)2 = 4x(25 − x)2 . Ta cần tìm x sao cho V đạt giá trị lớn nhất. Áp dụng bất đẳng thức giữa trung bình cộng và trung bình nhân cho tam V = 4x(25 − x)2 = 4x(25 − x)(25 − x) 3 2x + 25 − x + 25 − x ≤2 . 3 50 3 Từ đó suy ra V ≤ 2 3 . Dấu đẳng thức xảy ra khi và chỉ khi 2x = 25 − x = 25 − x. 25 25 Tức là, x = 3 . Nghĩa là, hình vuông bị cắt có cạnh là 3 (cm). Bài toán 3. Từ một mảnh giấy bìa có dạng hình chữ nhật kích thước 15 × 8 cm, người ta cắt ra từ bốn góc của hình chữ nhật bốn hình vuông bằng nhau. Mảnh giấy còn lại trông giống như một chữ thập, được gấp làm một cái hộp (không nắp). Hỏi rằng cạnh của bốn hình vuông bằng bao nhiêu để thể tích của chiếc hộp thu được là lớn nhất. 8 15 Chứng minh. Gọi x là độ dài cạnh của hình vuông. Khi đó chiếc hộp sẽ có kích thước là 15 − x − x, 8 − x − x, và x. Thể tích của chiếc hộp là V được tính theo x như sau V = x(15 − 2x)(8 − 2x). Ta cần tìm x sao cho V đạt giá trị lớn nhất. Khi chưa có công cụ đạo hàm, ta vẫn có thể tìm giá trị lớn nhất của V bằng cách sử dụng bất đẳng thức giữa trung bình cộng và trung bình nhân. 6
  • 7. cần phải tìm k, trong kx(15 − 2x) (8 − 2x) sao cho 15 − 2x = (8 − 2x) = kx, và 2 + 2 − k = 0. Từ đây, biểu diễn x theo k, , ta tìm k, bằng cách giải hệ phương trình 15 − 8 8 = , 2( 1 − ) k+2 2 + 2 − k = 0. 5 Giải hệ này cho ta = 2 , k = 7. Bây giờ ta áp dụng bất đẳng thức giữa trung bình cộng và trung bình nhân cho ba số, 2 V = x(15 − 2x)(8 − 2x) = .7x(15 − 2x)(20 − 5x) 35 3 2 7x + 15 − 2x + 20 − 5x ≤ . . 35 3 Từ đó, 3 2 35 2450 V≤ . = . 35 3 27 Đẳng thức xảy ra khi và chỉ khi 7x = 15 − 2x = 20 − 5x. 5 5 Giải hệ phương trình này cho ta x = 3 . Vậy cạnh hình vuông là 3 cm. Bài toán 4. Tìm tất cả các giá trị của x ≥ 0 sao cho 5 (3x + 1) − x = −2x2 + 3x + 1. 4 Giải. Điều kiện để phương trình có nghĩa là x ≤ 5 . Áp dụng bất đẳng thức giữa trung bình 4 cộng và trung bình nhân, ta có 5 1 5 1 5 −x=2 −x ≤ + − x. 4 4 4 4 4 Suy ra, vì 3x + 1 ≥ 0, ta có 5 3 (3x + 1) − x ≤ (3x + 1) −x . 4 2 Từ đây, ta có 3 −2x2 + 3x + 1 ≤ (3x + 1) −x . 2 7
  • 8. phải và rút gọn cho ta −2x2 + x + 1 ≥ 0, hay là (1 − x)(1 + 2x) ≤ 0. Đẳng thức xảy ra khi và chỉ khi x = 1 hoặc x = − 1 . Từ đó suy ra 2 x = 1 là giá trị cần tìm. Bài toán 5 (Bất đẳng thức Nesbitt). Cho ba số thực dương a, b, c. Chứng minh rằng a b c 3 + + ≥ . b+c c+a a+b 2 Chứng minh. Ta đã chứng minh được bất đẳng thức sau đây cho ba số thực dương a, b, c a+b+c √ 3 3 ≥ abc ≥ 1 . 3 a +1+ b 1 c Từ đó suy ra bất đẳng thức 1 1 1 ( a + b + c) + + ≥ 9. a b c Trở lại bất đẳng thức Nesbitt. Lưu ý rằng a b c a b c + + = +1 + +1 + +1 −3 b+c c+a a+b b+c c+a a+b 1 1 1 = ( a + b + c) + + −3 b+c c+a a+b 1 1 1 1 = ( a + b + b + c + c + a) + + −3 2 a+b b+c c+a 1 3 ≥ .9 − 3 = . 2 2 Phép chứng minh hoàn tất. Bài toán 6. Với x là một số thực dương, tìm giá trị nhỏ nhất của x3 + 1 f ( x) = . x Chứng minh. Ta viết lại f ( x) dưới dạng 1 1 1 f ( x) = + x2 = + + x2 . x 2x 2x Áp dụng bất đẳng thức giữa trung bình cộng và trung bình nhân cho ba số dương 1/(2x), 8
  • 9. , ta có 1 1 2x + 2x + x2 3 1 1 2 1 ≥ . .x = √ . 3 3 2x 2x 4 Từ đó suy ra 3 f ( x) ≥ √ . 3 4 Đẳng thức đạt được chỉ khi 1 1 = = x2 . 2x 2x 1 Tức là x = √ 3 . 2 Bài tập 4. Giải phương trình 4 x 3 x− + = 0. 2 8 Bài tập 5. Chứng minh rằng với mọi số thực không âm a, b, c, d, ta có a b c d + + + ≥ 2. b+c c+d d+a a+b 5 Bất đẳng thức giữa các lượng trung bình Bây giờ ta phát biểu và chứng minh bài toán bất đẳng thức giữa các lượng trung bình cho dãy các số thực a1 , a2 , · · · an . Định lý 5. Với n số thực không âm a1 , a2 , · · · , an , ta có a1 + a2 + · · · + an √ ≥ n a1 a2 · · · an . (3) n Chứng minh. Nhận xét rằng ta chỉ cần chứng minh bất đẳng thức với n số thực dương vì bất đẳng thức trở nên tầm thường khi một hay nhiều số bằng không. Phép chứng minh dưới đây được gọi là quy nạp kiểu Cauchy đề xuất năm 1821. Với n = 2, bất đẳng thức đúng, theo chứng minh đã trình bày trên. Giả thiết rằng bất đẳng thức đúng với n, ta sẽ chứng minh bất đẳng thức cũng đúng với n = 2k , k ≥ 1 nguyên. Áp dụng kết quả trường hợp n = 2, ta có √ √ √ a1 a2 + a3 a4 a1 + a2 + a3 + a4 4 a1 a2 a3 a4 ≤ ≤ . 2 4 Vậy bất đẳng thức (3) đúng với n = 4. Lại áp dụng kết quả của (3) trong trường hợp n = 2, ta có √ √ √ 4 a a a a + 4 a a a a 1 2 3 4 5 6 7 8 a + a2 + · · · + a8 8 a1 a2 · · · a8 ≤ ≤ 1 . 2 8 9
  • 10. bất đẳng thức (3) đúng với n, ta chứng minh nó cũng đúng với 2n. Xét 2n số không âm a1 , a2 , . . . , an , an+1 , an+2 , . . . , a2n . Ta có 1 1 a1 + a2 + · · · + an a + an+2 + · · · + a2n ( a1 + a2 + · · · + a2n ) = + n+1 2n 2 n n 1 √ √ ≥ ( n a1 a2 · · · an + n an+1 an+2 · · · a2n ) 2 √ √ √ ≥ n a1 a2 . . . an . n an+1 an+2 . . . a2n = 2n a1 a2 . . . a2n . Dấu đẳng thức xảy ra khi và chỉ a1 = a2 = · · · = a2n . Tiếp theo, ta giả thiết bất đẳng thức (3) đúng với n, rồi từ đó chứng minh nó cũng đúng với n − 1. Bước lùi này chính là để lấp chỗ trống giữa các lũy thừa của hai. Đặt a1 + a2 + · · · + an− 1 an = . n−1 Ta có 1 a1 + · · · + an− 1 n a1 + · · · + an− 1 a1 + · · · + an− 1 + ≥ a1 . . . an− 1 . , n n−1 n−1 hay a1 + a2 + · · · + an− 1 n a1 + · · · + an− 1 ≥ a1 . . . an− 1 . n−1 n−1 Nâng lũy thừa bậc n hai vế cho ta n a1 + a2 + · · · + an− 1 a1 + · · · an− 1 ≥ a1 a2 . . . an− 1 . n−1 n−1 a1 + a2 + ···+ an −1 Chia cả hai vế cho an = n−1 rồi lấy căn bậc n − 1 của hai vế vừa thu được cho ta a1 + a2 + · · · + an− 1 √ ≥ n −1 a1 a2 . . . an− 1 . n−1 Dấu đẳng thức xảy ra khi và chỉ khi a1 + a2 + · · · + an− 1 a1 = a2 = · · · = an = , n−1 hay là a1 = a2 = · · · = an−1 . Phép chứng minh hoàn tất. 1 n Bài toán 7. Chứng minh rằng dãy số un = 1 + n , n = 1, 2, . . ., là dãy số tăng. 10
  • 11. cần chứng minh n+1 n 1 1 1+ > 1+ . n+1 n Áp dụng bất đẳng thức giữa trung bình cộng và trung bình nhân cho n + 1 số, trong đó có n 1 số 1 + n và một số 1, ta có    1  n +1 1  ( 1 + 1 ) + · · · + ( 1 + 1 ) + 1 ≥ ( 1 + 1 ) · · · ( 1 + 1 )     , n+1  n n   n n  n số 1 + 1/n n số 1 + 1/n hay là n n+2 n +1 1 ≥ 1+ . n+1 n n+2 1 Lấy căn bậc n + 1 hai vế của bất đẳng thức trên và lưu ý n+1 = 1+ n+1 cho ta điều phải chứng minh. Bài toán 8. Chứng minh rằng nếu x ∈ R và p, q ∈ N, thì p q p q p2 q2 sin x cos x ≤ p+q . ( p + q) 2 Chứng minh. Vế trái của bất đẳng thức chứ hai nhân tử sin x và cos x. Ta biết rằng sin2 x + cos2 x = 1. Quan hệ giữa hai nhân tử có tổng bình phương là hằng số gợi ý cho ta áp dụng bất đẳng thức giữa trung bình cộng và trung bình nhân. Để rõ hơn, ta viết lại bất đẳng thức dưới dạng p p qq (sin2 x) p (cos2 x)q ≤ . ( p + q) p+q Áp dụng bất đẳng thức giữa trung bình cộng và trung bình nhân cho p số sin2 x/p và q số cos2 x/p, ta thu được sin 2 x sin2 x cos2 x cos2 x +···+ + +···+ p p q q p+q sin 2 x p cos2 x q ≥ . p+q p q Bây giờ sử dụng hằng đẳng thức sin 2 x + cos 2 x = 1, ta có p p qq (sin2 x) p (cos2 x)q ≤ , ( p + q) p+q từ đó ta có điều phải chứng minh. 11
  • 12. Cho hai số thực không âm a, b thỏa mãn điều kiện a2 + b2 = 1. Chứng minh rằng √ 3 3 ab + max {a, b} ≤ . 4 Chứng minh. Gọi vế trái của bất đẳng thức là f ( a, b). Không mất tính tổng quát, ta có thể giả sử a ≥ b. Khi đó f 2 ( a, b) = a2 (b + 1)2 . Áp dụng bất đẳng thức giữa trung bình cộng và trung bình nhân ta có f 2 ( a, b) = (1 − b2 )(1 + b)2 = 1 (3 − 3b)(1 + b)3 3 1 3 − 3b + 1 + b + 1 + b + 1 + b 4 27 ≤ = . 3 4 16 Từ đó suy ra điều phải chứng minh. Đẳng thức xảy ra khi và chỉ khi 3 − 3b = 1 + b và √ a2 + b2 = 1. Giải hệ này cho ta b = 1 , a = 23 , hoặc hoán vị của bộ này. 2 Bài toán 10. Cho a1 , a2 , · · · , an là các số thực dương thỏa mãn 2 1 1 1 1 ( a1 + a2 + · · · + an ) + +···+ ≤ +n . a1 a2 an 2 Chứng minh rằng max ( a1 , a2 , · · · , an ) ≤ 4 min ( a1 , a2 , · · · , an ) ai . Chứng minh. Đặt m = min ( a1 , a2 , . . . , an ) và M = max ( a1 , a2 , . . . , an ). Không mất tính tổng quát, ta giả sử m = a1 ≤ a2 ≤ · · · ≤ an = M. Đặt a = a2 +···+ an−1 thì ta có m ≤ a2 ≤ a ≤ n−2 an−1 ≤ M và a2 + · · · + an−1 = (n − 2) a. Theo bất đẳng thức giữa trung bình cộng và trung bình điều hòa 1 1 ( n − 2) 2 n−2 +···+ ≥ = . a2 an− 1 a2 + · · · + an− 1 a Suy ra 2 1 1 1 1 +n ≥ ( a1 + a2 + · · · + an ) + +···+ 2 a1 a2 an 1 n−2 1 ≥ ( m + ( n − 2) a + M ) + + m a M 1 1 (n − 2)(m + M ) (m + M ) + + ( m − 2) 2 + m M a 1 1 + ( n − 2) a + m M (m + M ) 2 (n − 2)(m + M ) mM = + ( n − 2) 2 + +a . mM mM a mM √ Theo bất đẳng thức giữa trung bình cộng và trung bình nhân, ta có a + a ≥ 2 mM, có 12
  • 13. ra tại m ≤ a = mM ≤ M. Ta suy ra 2 1 (m + M )2 2(n − 2)(m + M ) n+ ≥ + ( n − 2) 2 + √ . 2 mM mM Đặt t = √+ M m cho ta mM 2 1 n+ ≥ t2 + ( n − 2) 2 + 2( n − 2) t = ( t + n − 2) 2 , 2 từ đó suy ra 1 n+ ≥ t + n − 2. 2 5 Nghĩa là t ≤ 2 . Lưu ý, với n = 2, điều kiện bất đẳng thức có dạng 1 1 25 (m + M ) + ≤ . m M 4 Từ đó suy ra (4M − m)( M − 4m) ≤ 0. Vì 4M − m > 0, nên ta suy ra M − 4m ≤ 0, hay M ≤ 4m. Phép chứng minh hoàn tất. 6 Một số kỹ thuật cân bằng hệ số Ta tiếp tục nhấn mạnh đến một số kỹ thuật điều chỉnh hệ số trong khi áp dụng bất đẳng thức giữa trung bình cộng và trung bình nhân, Bài toán 11. Chứng minh rằng nếu a, b, c, d > 0, thì a b c d √ +√ +√ +√ ≥ 2. b2 + c2 + d2 c2 + d2 + a2 d2 + a2 + b2 a2 + b2 + c2 1 Chứng minh. Áp dụng bất đẳng thức giữa trung bình cộng và trung bình nhân, ta có 2 ( x + √ 1) ≥ x. Và do đó 2 2 2 b2 + c2 + d2 1 + b +c 2+d a ≤ . a2 2 Suy ra a 2a2 √ ≥ 2 . b2 + c2 + d2 a + b2 + c2 + d2 Cộng ba bất đẳng thức tương tự, ta suy ra điều phải chứng minh. Bài toán 12. Cho ba số thực không âm x, y và z thỏa mãn điều kiện x + y + z = 1. Tìm giá 13
  • 14. của biểu thức f = ( x2 − xy + y2 )( y2 − yz + z2 )( z2 − zx + x2 ). Chứng minh. Không mất tính tổng quát, ta giả sử x ≥ y ≥ z ≥ 0. Do đó x2 − xz + z2 ≤ x2 , y2 − yz + z2 ≤ y2 . Thành thử, f ≤ x2 y2 ( x2 − xy + y2 ) = v2 (u2 − 3v), trong đó u = x + y, v = xy, và s ≤ 1. Theo bất đẳng thức giữa trung bình cộng và trung bình nhân, ta có 3 4 3v 3v s2 4 v2 (s2 − 3v) ≤ + + − 3v = . 9 2 2 3 243 Đẳng thức xảy ra nếu và chỉ nếu  s   = 1, 3 v = 1 − 3v, 2  z = 0. 2 1 Tức là, đẳng thức xảy ra với bộ số ( x, y, z) là hoán vị của (0, 3 , 3 ). Bài toán 13. Chứng minh rằng nếu a, b, c, d ≥ 0, thì 1 ( a2 + b2 + c2 )(b2 + c2 + d2 )(c2 + d2 + a2 )(d2 + a2 + b2 ) ≤ ( a + b + c + d) 8 . 64 Chứng minh. Không mất tính tổng quát, ta giả sử a ≥ b ≥ c ≥ d. Do đó, 2 2 c d c d a2 + c2 + d2 ≤ a+ + , b2 + c2 + d2 ≤ b+ + , 2 2 2 2 và a2 + b2 + d2 ≤ a2 + b2 + c2 . Gọi L là vế trái của bất đẳng thức cần chứng minh. Vì vậy, c d 2 c d 2 L ≤ ( a2 + b2 + c2 ) 2 a + + b+ + . (4) 2 2 2 2 Theo bất đẳng thức giữa trung bình cộng và trung bình nhân, vế phải của R of (4) có thể được đánh giá như sau √ 1 2 c d c d 2 R≤ a + b2 + c2 + 2 a + + b+ + 8 2 2 2 2 2 2 2 1 c d c d c d c d ≤ a+ + + b+ + +2 a + + b+ + . 8 2 2 2 2 2 2 2 2 14
  • 15. x2 + y2 + 2xy = ( x + y)2 , ta thu được điều phải chứng minh. Bài toán 14. Cho các số thực không âm x, y, z thỏa mãn điều kiện x + y + z = 1. Tìm giá trị lớn nhất của f = 9xy + 10yz + 11zx. Chứng minh. Nhận xét rằng tồn tại một bộ ba số thực không âm p, q, r sao cho f = px( y + z) + qy( z + x) + rz( x + y), = ( p + q) xy + (q + r) yz + ( p + r) zx. Ta cần chọn p, q, r sao cho p + q = 9, q + r = 10, p + r = 11. Giải hệ phương trình này cho ta p = 4, q = 5, r = 6. Do đó ta có thể viết f dưới dạng f = 4x(1 − x) + 5y(1 − y) + 6z(1 − z). Lại viết f dưới dạng 2 2 2 15 1 1 1 f = − 4 x− +5 y− +6 z− . 4 2 2 2 Đặt | x − 1 | = a, | y − 1 | = b và | z − 1 | = c, thì từ điều kiện x + y + z = 1 ta sẽ có a + b + c ≥ 2 . 2 2 2 1 Bài toán chuyển về việc tìm giá trị nhỏ nhất của g = 4a2 + 5b2 + 6c2 , trong đó a, b, c là các số thực không âm thỏa mãn a + b + c ≥ 1 . 2 Ta chọn các số α , β, γ sao cho sau khi áp dụng bất đẳng thức giữa trung bình cộng và trung bình nhân, các đẳng thức xảy ra với cùng bộ a, b, c. Ta có a2 + α 2 ≥ 2aα , b2 + β2 ≥ 2bβ, c2 + γ 2 ≥ 2cγ . Suy ra g + 4α 2 + 5β2 + 6γ 2 ≥ 2(4α a + 5βb + 6γ c). Ta cần chọn α , β, và γ sao cho α = 5 β = 3 γ. Do đó 4 2 g 5 3 + α 2 + β2 + γ 2 ≥ 2α ( a + b + c) ≥ α . 4 4 2 15
  • 16. 15 5 3 f ≤ − 4 −α 2 − β2 − γ 2 + α . 4 4 2 Đẳng thức xảy ra với a + b + c = α + β + γ = α + 4 α + 2 α = 1. Do đó , α = 5 3 15 37 , β= 12 37 và 10 γ = 37 . Dẫn đến 15 4 5 60 f ≤ + 152 + .122 − 150 − . 4 1369 4 37 Tính toán đơn giản cho ta 582195 f ≤ ≈ 2, 8735. 202612 Phép chứng minh hoàn tất. Bài tập 6. Bốn số thực không âm x, y, z và t thỏa mãn điều kiện x + y + z + t = 1. Chứng minh bất đẳng thức xy yz zt tx ty zx 1 + + + + + ≤ . x+y+1 y+z+1 z+t+1 t+x+1 t+y+1 z+x+1 4 Bài tập 7. Cho ba số thực không âm a, b, c, d. Chứng minh rằng √ √ a2 + b2 + c2 a + b + c + ( 3 − 1) a2 + b2 + c2 − ab − bc − ca ≤ . 3 3 Bài tập 8. Bốn số thực không âm x, y, z, w thỏa mãn x + y + z + w = 1. Tìm giá trị lớn nhất của f = 17xy + 18xz + 19xw + 19yz + 20yw + 21zw. Tài liệu [1] Phan Đức Chính (1993) Bất đẳng thức, Nhà xuất bản Giáo dục, Việt Nam [2] Phạm Văn Thuận, Lê Vĩ (2009) Olympiad Inequalities, Intuitive Mathematics Press, Canada [3] USAMO (2009) http://www.unl.edu/amc/e-exams/e8-usamo/usamo.shtml 16